You are on page 1of 8

Paper 1

Regional Mathematical Olympiad 2013

December 1, 2013

1. Let ABC be an acute angled triangle. The circle with BC as diameter intersects AB and
AC again at P and Q, respectively. Determine BAC given that the orthocenter of triangle
AP Q lies on .
Solution. Let K denote the orthocenter of triangle AP Q. Since triangles ABC and AQP
are similar it follows that K lies in the interior of triangle AP Q.
Note that KP A = KQA = 90 A. Since BP KQ is a cyclic quadrilateral it follows that
BQK = 180 BP K = 90 A, while on the other hand BQK = BQA KQA =
A since BQ is perpendicular to AC. This shows that 90 A = A, so A = 45 .
2. Let f (x) = x3 + ax2 + bx + c and g(x) = x3 + bx2 + cx + a, where a, b, c are integers with
c 6= 0. Suppose that the following conditions hold:
(a) f (1) = 0;
(b) the roots of g(x) are squares of the roots of f (x).
Find the value of a2013 + b2013 + c2013 .
Solution. Note that g(1) = f (1) = 0, so 1 is a root of both f (x) and g(x). Let p and q be the
other two roots of f (x), so p2 and q 2 are the other two roots of g(x). We then get pq = c and
p2 q 2 = a, so a = c2 . Also, (a)2 = (p + q + 1)2 = p2 + q 2 + 1 + 2(pq + p + q) = b + 2b = b.
Therefore b = c4 . Since f (1) = 0 we therefore get 1 + c c2 + c4 = 0. Factorising, we
get (c + 1)(c3 c2 + 1) = 0. Note that c3 c2 + 1 = 0 has no integer root and hence
c = 1, b = 1, a = 1. Therefore a2013 + b2013 + c2013 = 1.
3. Find all primes p and q such that p divides q 2 4 and q divides p2 1.
Solution. Suppose that p q. Since q divides (p 1)(p + 1) and q > p 1 it follows that q
divides p + 1 and hence q = p + 1. Therefore p = 2 and q = 3.
On the other hand, if p > q then p divides (q 2)(q + 2) implies that p divides q + 2 or
q 2 = 0. This gives either p = q + 2 or q = 2. In the former case it follows that that q
divides (q +2)2 1, so q divides 3. This gives the solutions p > 2, q = 2 and (p, q) = (5, 3).
4. Find the number of 10-tuples (a1 , a2 , . . . , a10 ) of integers such that |a1 | 1 and
a21 + a22 + a23 + + a210 a1 a2 a2 a3 a3 a4 a9 a10 a10 a1 = 2 .
Solution. Let a11 = a1 . Multiplying the given equation by 2 we get
(a1 a2 )2 + (a2 a3 )2 + (a10 a1 )2 = 4 .
Note that if ai ai+1 =P
2 for some i = 1, . . . , 10, then aj aj+1 = 0 for all j 6= i which
contradicts the equality 10
i=1 (ai ai+1 ) = 0. Therefore ai ai+1 = 1 for exactly two values
of i in {1, 2, . . . , 10}, ai ai+1 = 1 for two other values of i and ai ai+1 = 0 for all other
8
values of i. There are 10
2 2 = 45 28 possible ways of choosing these values. Note
that a1 = 1, 0 or 1, so in total there are 3 45 28 possible integer solutions to the given
equation.
1

www.examrace.com

Paper 1

Regional Mathematical Olympiad 2013

December 1, 2013

5. Let ABC be a triangle with A = 90 and AB = AC. Let D and E be points on the segment
BC such that BD : DE : EC = 3 : 5 : 4. Prove that DAE = 45 .
Solution. Rotating the configuraiton about A by 90 , the point B goes to the point C. Let
P denote the image of the point D under this rotation. Then CP = BD and ACP =
ABC = 45 , so ECP is a right-angled triangle with CE : CP = 4 : 3. Hence P E = ED.
It follows that ADEP is a kite with AP = AD and P E = ED. Therefore AE is the angular
bisector of P AD. This implies that DAE = P AD/2 = 45 .
6. Suppose that m and n are integers such that both the quadratic equations x2 + mx n = 0
and x2 mx + n = 0 have integer roots. Prove that n is divisible by 6.
Solution. Let a be an integer. If a is not divisible by 3 then a2 1 (mod 3), i.e., 3 divides
a2 1, and if a is odd then a2 1 (mod 8), i.e., 8 divides a2 1.
Note that the discriminants of the two quadratic polynomials are both squares of integers.
Let a and b be integers such that m2 4n = a2 and m2 + 4n = b2 . Therefore 8n = b2 a2
and 2m2 = a2 + b2 . If 3 divides m then 3 divides both a and b, so 3 divides n. On the other
hand if 3 does not divide m then 3 does not divide a or b. Therefore 3 divides b2 a2 and
hence 3 divides n.
If m is odd, then so is a, and therefore 4n = m2 a2 is divisible by 8, so n is even. On
the other hand, if m is even then both a and b are even. Further (m/2)2 n = (a/2)2 and
(m/2)2 + n = (b/2)2 , so (b a)/2 is even. In particular, n = (b2 a2 )/4 is even.
?

www.examrace.com

Paper 2

Regional Mathematical Olympiad 2013

December 1, 2013

1. Prove that there do not exist natural numbers x and y, with x > 1, such that
x7 1
= y5 + 1 .
x1
Solution. Simple factorisation gives y 5 = x(x3 + 1)(x2 + x + 1). The three factors on the
right are mutually coprime and hence they all have to be fifth powers. In particular, x = r5
for some integer r. This implies x3 + 1 = r15 + 1, which is not a fifth power unless r = 1 or
r = 0. This implies there are no solutions to the given equation.
2. In a triangle ABC, AD is the altitude from A, and H is the orthocentre. Let K be the centre
of the circle passing through D and tangent to BH at H. Prove that the line DK bisects AC.
Solution. Note that KHB = 90 . Therefore KDA = KHD = 90 BHD =
HBD = HAC. On the other hand, if M is the midpoint of AC then it is the circumcenter
of triangle ADC and therefore M DA = M AD. This proves that D, K, M are collinear
and hence DK bisects AC.
3. Consider the expression
20132 + 20142 + 20152 + + n2 .
Prove that there exists a natural number n > 2013 for which one can change a suitable
number of plus signs to minus signs in the above expression to make the resulting expression
equal 9999.
Solution. For any integer k we have
k 2 + (k + 1)2 + (k + 2)2 (k + 3)2 = 4 .
Note that 9999 (20132 + 20142 + 20152 + 20162 + 20172 ) = 4m for some positive integer
m. Therefore, it follows that
9999 =(20132 + 20142 + 20152 + 20162 + 20172 )
m
X

+
(4r + 2014)2 + (4r + 2015)2 + (4r + 2016)2 (4r + 2017)2 .
r=1

4. Let ABC be a triangle with A = 90 and


AB = AC. Let D and E be points on the segment
BC such that BD : DE : EC = 1 : 2 : 3. Prove that DAE = 45 .
Solution. Rotating the configuraiton about A by 90 , the point B goes to the point C. Let
P denote the image of the point D under this rotation. Then CP
= BD and ACP =

ABC = 45 , so ECP is a right-angled triangle with CE : CP = 3 : 1. Hence P E = ED.


It follows that ADEP is a kite with AP = AD and P E = ED. Therefore AE is the angular
bisector of P AD. This implies that DAE = P AD/2 = 45 .

www.examrace.com

Paper 2

Regional Mathematical Olympiad 2013

December 1, 2013

5. Let n 3 be a natural number and let P be a polygon with n sides. Let a1 , a2 , . . . , an be the
lengths of the sides of P and let p be its perimeter. Prove that
a1
a2
an
+
+ +
< 2.
p a1 p a2
p an
Solution. If r and s are positive real numbers such that r < s then r/s < (r + x)/(s + x)
for any positive real x. Note that, by triangle inequality, ai < p ai , so
ai
2ai
<
,
p ai
p
for all i = 1, , 2 . . . , n. Summing this inequality over i we get the desired inequality.
6. For a natural number n, let T (n) denote the number of ways we can place n objects of weights
1, 2, . . . , n on a balance such that the sum of the weights in each pan is the same. Prove that
T (100) > T (99).
Solution. Let S(n) denote the collection of subsets A of X(n) = {1, 2, . . . , n} such that
the sum of the elements of A equals n(n + 1)/4. Then the given inequality is equivalent to
|S(100)| > |S(99)|. We shall give a map f : S(99) S(100) which is one-to-one but not
onto. Note that this will prove the required inequality.
Suppose that A is an element of S(99). If 50 A then define f (A) = (A \ {50}) {100}.
Otherwise, define f (A) = A {50}. If A and B are elements of S(99) such that f (A) = f (B)
then either 50 belongs to both these sets or neither of these sets. In either of the cases we
have A = B. Therefore f is a one-to-one function.
Note that every element in the range of f contains exactly one of 50 and 100. Let Bi =
{i, 101 i}. Then B1 B2 B24 B50 is an element of S(100). Clearly, this is not in the
range of f . Thus f is not an onto function.
??

www.examrace.com

Paper 3

Regional Mathematical Olympiad 2013

December 1, 2013

1. Find the number of eight-digit numbers the sum of whose digits is 4.


Solution. We need to find the number of 8-tuples (a1 , a2 , . . . , a8 ) of non-negative integers
such that a1 1 and a1 + a2 + + a8 = 4. If a1 = 1, then there are three possibilities: either
exactly three among a2 , a3 , . . . , a7 equal 1 and the rest equal zero, or five of them are zero
and the other two equal 1 and 2, or six of them are zero and the other
 equals 3. In the first
case, there are 73 = 35 such 8-tuples, in the second case there are 72 2 = 42 such 8-tuples
and in the third case there are 7 such 8-tuples. If a1 = 2 then either six of a2 , a3 , . . . , a7 are
zero and the other equals two, or five of them are zero and the remaining two both
 equal 1.
In the former case, there are 7 such 8-tuples and in the latter case there are 72 = 21 such
8-tuples. If a1 = 3 then exactly six of a2 , a3 , . . . , a7 are zero and the other equals one. There
are 7 such 8-tuples. Finally, there is one 8-tuple in which a1 = 4. Thus, in total, there are
120 such 8-tuples.
2. Find all 4-tuples (a, b, c, d) of natural numbers with a b c and a! + b! + c! = 3d .
Solution. Note that if a > 1 then the left-hand side is even, and therefore a = 1. If b > 2
then 3 divides b! + c! and hence 3 does not divide the left-hand side. Therefore b = 1 or b = 2.
If b = 1 then c! + 2 = 3d , so c < 2 and hence d = 1. If b = 2 then c! = 3d 3. Note that
d = 1 does not give any solution. If d > 1 then 9 does not divide c!, so c < 6. By checking
the values for c = 2, 3, 4, 5 we see that c = 3 and c = 4 are the only two solutions. Thus
(a, b, c, d) = (1, 1, 1, 1), (1, 2, 3, 2) or (1, 2, 4, 3).
3. In an acute-angled triangle ABC with AB < AC, the circle touches AB at B and passes
through C intersecting AC again at D. Prove that the orthocentre of triangle ABD lies on
if and only if it lies on the perpendicular bisector of BC.
Solution. Note that ADB = B and hence triangles ADB and ABC are similar. In
particular, ABD is an acute-angled triangle. Let H denote the orthocenter of triangle ABD.
Then BHD = 180 A.
Suppose that H lies on . Since AB < AC the point D lies on the segment AC and
C = 180 BHD = A. Therefore BH is the perpendicular bisector of AC. Hence
HBC = ABC = HCB, so H lies on the perpendicular bisector of BC.
Conversely, suppose that H lies on the perpendicular bisector of BC. Then HCB =
HBC = 90 C. Since ABD = C it follows that HDB = 90 C. Since
HCB = HDB we have that H lies on .
4. A polynomial is called a Fermat polynomial if it can be written as the sum of the squares
of two polynomials with integer coefficients. Suppose that f (x) is a Fermat polynomial such
that f (0) = 1000. Prove that f (x) + 2x is not a Fermat polynomial.
Solution. Let p(x) be a Fermat polynomial such that p(0) is divisible by 4. Suppose that
p(x) = g(x)2 + h(x)2 where g(x) and h(x) are polynomials with integer coefficients. Therefore
g(0)2 + h(0)2 is divisble by 4. Since g(0) and h(0) are integers, their squares are either
1 (mod 4) or 0 (mod 4). It therefore follows that g(0) and h(0) are even. Therefore the
1

www.examrace.com

Paper 3

Regional Mathematical Olympiad 2013

December 1, 2013

coefficents of x in g(x)2 and in h(x)2 are both divisible by 4. In particular, the coefficient of
x in a Fermat polynomial p(x), with p(0) divisible by 4, is divisible by 4. Thus if f (x) is a
Fermat polynomial with f (0) = 1000 then f (x) + 2x cannot be a Fermat polynomial.

5. Let ABC be a triangle which it not right-angled. Define a sequence of triangles Ai Bi Ci ,


with i 0, as follows: A0 B0 C0 is the triangle ABC; and, for i 0, Ai+1 , Bi+1 , Ci+1 are the
reflections of the orthocentre of triangle Ai Bi Ci in the sides Bi Ci , Ci Ai , Ai Bi , respectively.
Assume that Am = An for some distinct natural numbers m, n. Prove that A = 60 .
Solution. The statement of the problem as stated is not correct. We give below the reason,
and we shall also give the condition under which the statement becomes true.
Let P, Q, R denote the reflections of H with respect BC, CA, AB, respectively. Then P, Q, R
lie on the circumcircle of the triangle. If ABC is an acute-angled triangle then QP R =
QP A + RP A = QCA + RBA = 180 2A. Similarly, if A is obtuse then we get
QP R = 2A 180 . Therefore, for example, if A = 180 /7 and BC = 540 /7 then we
get that A3 = 180 /7 = A0 . Therefore the statement of the problem is not correct.
However, the statement is correct provided all the triangles Ai Bi Ci are acute-angled. Under
this assumption we give below a proof of the statement.
Let , , denote the angles of T0 . Let fk (x) = (2)k x ((2)k 1)60 . We claim that the
angles of Tk are fk (), fk () and fk (). Note that this claim is true for k = 0 and k = 1. It
is easy to check that fk+1 (x) = 180 2fk (x), so the claim follows by induction.
If Tm = Tn , then fm () = fn (), so ((2)m (2)n ) = 60 ((2)m (2)n ). Therefore,
since m 6= n, it follows that = 60 .
6. Let n 4 be a natural number. Let A1 A2 An be a regular polygon and X = {1, 2, . . . , n}.
A subset {i1 , i2 , . . . , ik } of X, with k 3 and i1 < i2 < < ik , is called a good subset
if the angles of the polygon Ai1 Ai2 Aik , when arranged in the increasing order, are in an
arithmetic progression. If n is a prime, show that a proper good subset of X contains exactly
four elements.
Solution. We note that every angle of Ai1 Ai2 Aik is a multiple of /n. Suppose that these
angles are in an arithmetic progression. Let r and s be non-negative integers such that r/n
is the smallest angle in this progression and s/n is the common difference. Then we have

(rk + sk(k 1)/2) = (k 2) .


n
Therefore rk + sk(k 1)/2 = (k 2)n. Suppose that k is odd. Then k divides the left-hand
side and k is coprime to k 2. Therefore k divides n. On the other hand if k is even then
k/2 is coprime to (k 2)/2 and hence k divides 4n. If n is prime and k < n then it follows
that k divides 4. Since k > 2, we have proved that k = 4.
? ? ?

www.examrace.com

Paper 4

Regional Mathematical Olympiad 2013

December 1, 2013

1. Let be a circle with centre O. Let be another circle passing through O and intersecting
at points A and B. A diameter CD of intersects at a point P different from O. Prove
that
AP C = BP D .
Solution. Suppose that A0 is a point on such that A0 P C = BP D. Then the segments
OA0 and OB subtends same angle in the respective minor arcs, so OA0 = OB. This shows
that A lies on and hence A0 = A. This proves that AP C = BP D.
2. Determine the smallest prime that does not divide any five-digit number whose digits are in
a strictly increasing order.
Solution. Note that 12346 is even, 3 and 5 divide 12345, and 7 divides 12348. Consider a 5
digit number n = abcde with 0 < a < b < c < d < e < 10. Let S = (a + c + e) (b + d). Then
S = a + (c b) + (e d) > a > 0 and S = e (d c) (b a) < e 10, so S is not divisible
by 11 and hence n is not divisible by 11. Thus 11 is the smallest prime that does not divide
any five-digit number whose digits are in a strictly increasing order.
3. Given real numbers a, b, c, d, e > 1 prove that
a2
b2
c2
d2
e2
+
+
+
+
20 .
c1 d1 e1 a1 b1
Solution. Note that (a 2)2 0 and hence a2 4(a 1). Since a > 1 we have
By applying AM-GM inequality we get

a2
4.
a1

s
a2
b2
c2
d2
e2
a2 b2 c2 d2 e2
+
+
+
+
55
20 .
c1 d1 e1 a1 b1
(a 1)(b 1)(c 1)(d 1)(e 1)

1
1
4. Let x be a non-zero real number such that x4 + 4 and x5 + 5 are both rational numbers.
x
x
1
Prove that x + is a rational number.
x
Solution. For a natural number k let Tk = xk + 1/xk . Note that T4 T2 = T2 + T6 and
T8 T2 = T10 + T6 . Therefore T2 (T8 T4 + 1) = T10 . Since T2k = Tk2 + 2 it follows that T8 , T10
are rational numbers and hence T2 , T6 are also rational numbers. Since T5 T1 = T4 + T6 it
follows that T1 is a rational number.
5. In a triangle ABC, let H denote its orthocentre. Let P be the reflection of A with respect to
BC. The circumcircle of triangle ABP intersects the line BH again at Q, and the circumcircle
of triangle ACP intersects the line CH again at R. Prove that H is the incentre of triangle
P QR.

www.examrace.com

Paper 4

Regional Mathematical Olympiad 2013

December 1, 2013

Solution. Since RACP is a cyclic quadrilateral it follows that RP A = RCA = 90 A.


Similarly, from cyclic quadrilateral BAQP we get QP A = 90 A. This shows that P H
is the angular bisector of RP Q.
We next show that R, A, Q are collinear. For this, note that BP C = A. Since BHC =
180 A it follows that BHCP is a cyclic quadrilateral. Therefore RAP + QAP =
RCP + QBP = 180 . This proves that R, A, Q are collinear.
Now QRC = ARC = AP C = P AC = P RC. This proves that RC is the angular
bisector of P RQ and hence H is the incenter of triangle P QR.
6. Suppose that the vertices of a regular polygon of 20 sides are coloured with three colours
red, blue and green such that there are exactly three red vertices. Prove that there are three
vertices A, B, C of the polygon having the same colour such that triangle ABC is isosceles.
Solution. Since there are exactly three vertices, among the remaining 17 vertices there are
nine of them of the same colour, say blue. We can divide the vertices of the regular 20-gon
into four disjoint sets such that each set consists of vertices that form a regular pentagon.
Since there are nine blue points, at least one of these sets will have three blue points. Since
any three points on a pentagon form an isosceles triangle, the statement follows.

www.examrace.com

You might also like